3
$\begingroup$

Let $A_n$ be the alternating group of $\{1,2,\cdots,n\}$.

(1). What is the cohomology ring $$ H^*(A_4;\mathbb{Z}/3) $$ and its Steenrod operation $P^i$'s?

(2). Are there general results about the cohomology ring $$ H^*(A_{p+1};\mathbb{Z}/p) $$ for general primes $p\geq 3$?

(3). What is the cohomology ring $$ H^*(A_n;\mathbb{Z}_2) $$ for $n\geq 4$ (we can impose conditions on $n$)?

Any references?

$\endgroup$
3
  • 2
    $\begingroup$ For odd prime $p$, he $p$-sylow subgroup of $A_{p+1}$ is $Z/p$, so they are a direct summand of $H^*(BZ/p.Z/p)$. It shouldn't be too hard to determine. $\endgroup$
    – user43326
    Oct 4, 2015 at 6:34
  • $\begingroup$ Alternating groups have homological stability, and the stable homology is computable via scanning, eg arxiv.org/abs/1306.6896. $\endgroup$
    – skupers
    Oct 4, 2015 at 15:05
  • $\begingroup$ @skupers OP is asking for the homology of individual $A_m$. $\endgroup$
    – user43326
    Oct 5, 2015 at 8:05

2 Answers 2

1
$\begingroup$

An easier way for the first case

Consider $P$, a $3$-Sylow subgroup of $A_4$. For example, take the cyclic group generated by the cyclic permutation (123). It is self normalizing, so the double coset formula for the compositions $BP\rightarrow BA_4 \stackrel{tr}{\rightarrow}BP$ where $tr$ denotes the transfer, reduces to the identity. Therefore $BP$ and $BA_4$ are homotopy equivalent at the prime 3.

For the second case, let's start with the review of the well-known computation of $H^*(B\Sigma _p;Z/p)$. Again as in the above, denote by $P$ the cyclic group generated by the cyclic permutation $(12\cdots p)$. Denote by $N$ the group of affine automorphisms of $Z/p$, considered as a subgroup of the permutation of $\{1,\ldots ,p\}$. Then $N$ is the normalizer of $P$ in $\Sigma _p$, with $N/P\cong Gl_1(Z/p)$ with the canonical action on $Z/p$. Now the double coset formula for the compositions $BP\stackrel{Bi}{\rightarrow} B\Sigma _p \stackrel{tr}{\rightarrow}BP$ says that in mod $p$ cohomology, this composition is the sum of maps induced by the multiplication by $s\in Gl_1(Z/p)$. In other words, we have, $$Bi^*tr^*(\beta ^{\epsilon}x^i)=\Sigma _{s\in Gl_1(Z/p)}\beta ^{\epsilon}s^ix^i\in H^*(BZ/p,Z/p)\cong Z/p[x]\otimes \Lambda _{Z/p}(\beta x).$$ The little Theorem of Fermat then shows that this is identity if $p-1\mid i$, $0$ otherwise. Therefore we get $$H^*(B\Sigma _p,Z/p)\cong Z/p[x^{p-1}]\otimes \Lambda _{Z/p}(\beta x^{p-1})\subset H^*(BZ/p,Z/p).$$

Now, if we replace $\Sigma _p$ with $A_{p+1}$, what changes is the normalizer. It is easy to see that $$N_{A_{p+1}}(P)=A_{p+1}\cap \Sigma _p.$$ We also see that the generator of $Gl_1(Z/p)$ acts on $P$ as a cyclic permutation of length $p-1$, thus its an odd permutation. This means in the appropriate double coset formula, the only half of $s's$ get involved (squares in $Gl_1(Z/p)$), and we arrive at the conclusion $$H^*(BA _{p+1},Z/p)\cong Z/p[x^\frac{p-1}{2}]\otimes \Lambda _{Z/p}(\beta x^\frac{p-1}{2})\subset H^*(BZ/p,Z/p).$$ When $p=3$, this reduces to the first case treated in the above.

$\endgroup$
8
$\begingroup$

Regarding your first question, you have a group extension $$1\rightarrow (\mathbb{Z}/2)^2\rightarrow A_4\rightarrow \mathbb{Z}/3\rightarrow 1.$$ Then we can use the Lyndon-Hochschild-Serre spectral sequence $$E_2^{p,q}=H^p(\mathbb{Z}/3,H^q((\mathbb{Z}/2)^2,\mathbb{Z}/3))\implies H^{p+q}(A_4,\mathbb{Z}/3)$$ a priori the $E_2$-page involves local coefficient cohomology. But what you get, by triviality of $H^q((\mathbb{Z}/2)^2,\mathbb{Z}/3)$ is that the projection map $A_4\rightarrow \mathbb{Z}/3$ induces an isomorphism of unstable $\mathcal{A}_3$-algebra: $$H^{*}(A_4,\mathbb{Z}/3)\cong H^{*}(\mathbb{Z}/3,\mathbb{Z}/3).$$ And $H^*(\mathbb{Z}/3,\mathbb{Z}/3)$ is isomorphic to the tensor algebra $\Lambda(x)\otimes \mathbb{Z}/3[y]$ where $x$ of degree 1 and $y$ of degree $2$ moreover $y=\beta(x)$.

And you can also use the same spectral sequence to compute $H^*(A_4,\mathbb{Z}/2)$. What you get is an isomorphism of algebras $$H^*(A_4,\mathbb{Z}/2)\cong H^0(\mathbb{Z}/3,H^*((\mathbb{Z}/2)^2,\mathbb{Z}/2))\cong \mathbb{Z}/2[u,v]^{\mathbb{Z}/3}$$ where $u$ and $v$ are of degree 1.

EDIT: If you want to compute the cohomology algebra of $H^*(\mathbb{Z}/3,\mathbb{Z}/3)$. You can use the fact that $$B\mathbb{Z}/3\simeq L(\infty,3)$$ where $L(\infty,3)=\bigcup_n L(n,3)$ is a limit of Lens spaces. And then use the fibration $$S^1\rightarrow L(\infty,3)\rightarrow \mathbb{C}P^{\infty}.$$ Playing with the spectral sequence of this fibration you get the claimed results: $$H^*(\mathbb{Z}/3,\mathbb{Z}/3)\cong H^*(S^1,\mathbb{Z}/3)\otimes H^*(\mathbb{C}P^{\infty},\mathbb{Z}/3)\cong \Lambda(x)\otimes \mathbb{Z}/3[y].$$ In order to compute $\beta(x)$,just look at this spectral sequence over the integers and observe that the differential $$E_2^{0,1}=\mathbb{Z}\langle x \rangle\rightarrow E_2^{2,0}=\mathbb{Z}\langle y \rangle$$ is such that $dx=3y$. Now the action of $\mathcal{A}_3$ is completely determined by the Cartan relation and the fact that:

  • $P^k(x)$ when $k>0$ by unstability,
  • $P^1(y)=y^3$ (because $y$ is of degree 2) and $P^k(y)=0$ when $k>1$.
$\endgroup$

Your Answer

By clicking “Post Your Answer”, you agree to our terms of service and acknowledge you have read our privacy policy.

Not the answer you're looking for? Browse other questions tagged or ask your own question.